Переход от 4-потенциала к E и B [закрыто]

В моих конспектах лекций есть шаг, которому я не могу следовать:

я 2 [ γ мю , γ ν ] ( мю А ν ν А мю ) "=" о мю ν Ф мю ν "=" я α Е о Б

Может ли кто-нибудь помочь мне с этим шагом? γ мю – дирак-гамма-матрицы, A – электромагнитный 4-потенциал. α я представляет собой блочную матрицу 2x2 с матрицами Паули о я в недиагональных элементах.

Тот же самый шаг можно найти в Itzykson - Quantum Field Theory: Page 66 Eq. 2-73. Без дальнейших объяснений.

Ответы (2)

Вы не определяете о . Вроде как и у Ицыксона/Зубера, вернее, он там определен как вектор с матрицами Паули в качестве компонентов (если не ошибаюсь). Однако эти матрицы имеют размер 2x2, тогда как все остальные матрицы в вашем уравнении имеют размер 4x4. Поэтому такое определение кажется несовместимым с вашим уравнением. Я считаю, что вы должны определить новые матрицы 4x4 о я , я "=" 1 , 2 , 3 (как и все дальнейшие латинские индексы), как о я "=" ϵ я Дж к о Дж к (здесь я могу опустить некоторые константы). Затем α я совпадает с [\gamma_0,gamma_i] (с точностью до константы), что совпадает с о 0 я (с точностью до константы). Затем вы используете Б я "=" ϵ я Дж к Ф Дж к (опять же, с точностью до константы). В приведенном выше я игнорирую различие между верхними и нижними индексами.

Спасибо за ответ, Ахметели! В своих конспектах лекций я нашел определение вектора о где-то: о я "=" я 4 ϵ я Дж к [ γ Дж , γ к ] ...но это было определено в совершенно другом контексте. Не уверен, что это действительно то же самое о .
Также, α называется скоростью в другой точке... это поможет? Извините за путаницу!
@Mechanix: это совпадает с определением, которое я предложил (опять же, с точностью до константы) - см. определение о Дж к .

Если вы определяете свои гамма-матрицы как блочные матрицы:

γ 0 "=" [ 1 0 0 1 ] ; γ я "=" [ 0 о я о я 0 ]

Затем вы также можете определить блочные матрицы о мю ν "=" я 2 ( γ мю γ ν γ ν γ мю ) . И учитывая скалярный потенциал ( А 0 , А 1 , А 2 , А 3 ) можно определить тензор Фарадея Ф мю ν "=" мю А ν ν А мю .

Итак, первое равенство

я 2 [ γ мю , γ ν ] ( мю А ν ν А мю ) "=" о мю ν Ф мю ν
следует по определению. И то, что это действительно говорит, является матричным уравнением. Для каждого мю и ν есть сложная матрица 4x4 о мю ν и действительный скаляр Ф мю ν , масштабируйте матрицу с помощью этого скаляра, а затем возьмите каждую такую ​​матрицу и сложите их все, вы получите матрицу. То же самое для левой части, и на самом деле это одна и та же матрица, один и тот же скаляр и одна и та же сумма.

Что касается второго равенства, нам нужно связать Ф к Е и Б . У нас есть Ф 0 я "=" Е я и Б я "=" 1 2 ϵ я Дж к Ф Дж к , где Ф Дж к "=" Ф Дж к численно. Итак, теперь мы можем разбить эту сумму на две части. Сначала что, если мю "=" 0 , затем ν 0 (иначе мы получим нулевую матрицу, масштабированную нулевым скаляром), поэтому давайте назовем ν к я чтобы указать, что это 1,2 или 3. Тогда мы имеем (для каждого фиксированного i):

о 0 я Ф 0 я "=" я 2 ( γ 0 γ я γ я γ 0 ) Е я "=" я 2 ( [ 1 0 0 1 ] [ 0 о я о я 0 ] [ 0 о я о я 0 ] [ 1 0 0 1 ] ) Е я .
Таким образом, выполняя вычисление блочной матрицы (опять же для каждого фиксированного i), мы получаем:
о 0 я Ф 0 я "=" я 2 ( [ 0 о я о я 0 ] [ 0 о я о я 0 ] ) Е я "=" я [ 0 о я о я 0 ] Е я "=" я α я Е я .

Так что это тот случай, когда мю "=" 0 и ν "=" я . Точно так же, если мю "=" я и ν "=" 0 . (А если серьезно, я думаю, что ваш результат может отличаться в два раза, так как оба эти термина входят в сумму о мю ν Ф мю ν .) Далее можно рассмотреть случай, когда мю "=" я и ν "=" Дж но на этот раз вы используете результат коммутатора для [ о я , о Дж ] и что вы будете делать, это выяснить, что о должно быть, так как никто не сказал. (Этот термин не используется в моих книгах по квантовой теории поля.) Но вы можете определить его таким, каким он вам нужен, просто вычислив левую часть. Вычисление левой части, где вы знаете, что это такое, должно дать вам выражение, которое вы можете сделать похожим на правую часть (за исключением, может быть, этого надоедливого множителя два).

Спасибо за ваш ответ ... но как я могу показать, что правая часть (часть с E и B) эквивалентна?
Большое спасибо! Могу ли я использовать то же самое α я для мю "=" я и ν "=" 0 ?
я заканчиваю с ϵ я Дж к [ о к 0 0 о к ] Ф я Дж "=" я α я Е я я 4 ϵ я Дж к [ о к 0 0 о к ] Б я Как я могу справиться α я ?